Question stems Flashcards
The argument makes which one of the following assumptions?
Necessary Assumption question
The argument assumes
Necessary Assumption
Which one of the following is an assumption on which the argument relies?
Necessary Assumption
Which one of the following is an assumption that the fundraiser’s argument depends on?
Necessary Assumption
Which one of the following is an assumption required by the argument?
Necessary Assumption
Which one of the following is an assumption made by the argument?
Necessary Assumption
Which one of the following is an assumption necessary for the critic’s conclusion to be properly drawn?
Necessary Assumption
Which one of the following most accurately expresses the main conclusion of the economist’s argument?
Main Point Question
Which one of the following most accurately expresses the conclusion of the surrealist’s argument?
Main Point Question
Which one of the following most accurately expresses the conclusion of the argument as a whole?
Main Point Question
The consultant’s statements, if true, most strongly support which one of the following?
Most Strongly Supported (MSS)
Which one of the following conclusions is most strongly supported by the information above?
Most Strongly Supported (MSS)
Which one of the following is most strongly supported by the information above?
Most Strongly Supported (MSS)
Which one of the following is most reasonably supported by the information above?
Most Strongly Supported (MSS)
Which one of the following is most strongly supported by the statements above, if they are true?
Most Strongly Supported (MSS)
Which one of the following, if true, is the logically strongest counter that Albert can make to Erin’s argument?
Weakening Question
Which one of the following statements, if true, most weakens the argument?
Weakening Question
Which one of the following, if true, most weakens the argument?
Weakening Question
Which one of the following, if true, most seriously undermines the conclusion drawn in the argument above?
Weakening Question
Which one of the following, if true, would most weaken the argument in the newspaper article?
Weakening Question
Which one of the following, if true, most strengthens the argument?
Strengthening Question
Which one of the following, if true, would most strengthen the archaeologist’s reasoning?
Strengthening Question
Which one of the following, if true, lends the most support to the psychologist’s conclusion?
Strengthening Question
Which one of the following, if true, most strengthens the economist’s reasoning?
Strengthening Question
If the statements above are true, which one of the following must be true?
MUST BE TRUE (MBT)
Which one of the following can be properly inferred from the statements above?
MUST BE TRUE (MBT)